JCECE Engineering JCECE Engineering Solved Paper-2004

  • question_answer
    If \[\overrightarrow{\mathbf{a}},\,\,\overrightarrow{\mathbf{b}},\,\,\overrightarrow{\mathbf{c}}\]are the non-coplanar vectors, then the value of\[\frac{\overset{\to }{\mathop{\mathbf{a}}}\,\cdot (\overset{\to }{\mathop{\mathbf{b}}}\,\times \overset{\to }{\mathop{\mathbf{c}}}\,)}{(\overset{\to }{\mathop{\mathbf{c}}}\,\times \overset{\to }{\mathop{\mathbf{a}}}\,)\cdot \overset{\to }{\mathop{\mathbf{b}}}\,}+\frac{\overset{\to }{\mathop{\mathbf{b}}}\,(\overset{\to }{\mathop{\mathbf{a}}}\,\times \overset{\to }{\mathop{\mathbf{c}}}\,)}{\overset{\to }{\mathop{\mathbf{c}}}\,\cdot (\overset{\to }{\mathop{\mathbf{a}}}\,\times \overset{\to }{\mathop{\mathbf{b}}}\,)}\]is:

    A) \[1\]     

    B) \[2\]

    C) \[0\]                                     

    D)  None of these

    Correct Answer: C

    Solution :

    Key Idea: If \[\overset{\to }{\mathop{\mathbf{a}}}\,,\,\,\overset{\to }{\mathop{\mathbf{b}}}\,\] and \[\overset{\to }{\mathop{\mathbf{c}}}\,\] are non-coplanar vectors, then\[[\overset{\to }{\mathop{\mathbf{a}}}\,\overset{\to }{\mathop{\mathbf{b}}}\,\overset{\to }{\mathop{\mathbf{c}}}\,]\ne 0\]. \[\therefore \]  \[\frac{\overset{\to }{\mathop{\mathbf{a}}}\,\cdot (\overset{\to }{\mathop{\mathbf{b}}}\,\times \overset{\to }{\mathop{\mathbf{c}}}\,)}{(\overset{\to }{\mathop{\mathbf{c}}}\,\times \overset{\to }{\mathop{\mathbf{a}}}\,)\cdot \overset{\to }{\mathop{\mathbf{b}}}\,}+\frac{\overset{\to }{\mathop{\mathbf{b}}}\,\cdot (\overset{\to }{\mathop{\mathbf{a}}}\,\times \overset{\to }{\mathop{\mathbf{c}}}\,)}{\overset{\to }{\mathop{\mathbf{c}}}\,\cdot (\overset{\to }{\mathop{\mathbf{a}}}\,\times \overset{\to }{\mathop{\mathbf{b}}}\,)}\]                 \[=\frac{[\overset{\to }{\mathop{\mathbf{a}}}\,\overset{\to }{\mathop{\mathbf{b}}}\,\overset{\to }{\mathop{\mathbf{c}}}\,]}{[\overset{\to }{\mathop{\mathbf{c}}}\,\overset{\to }{\mathop{\mathbf{b}}}\,\overset{\to }{\mathop{\mathbf{a}}}\,]}+\frac{[\overset{\to }{\mathop{\mathbf{b}}}\,\overset{\to }{\mathop{\mathbf{a}}}\,\overset{\to }{\mathop{\mathbf{c}}}\,]}{[\overset{\to }{\mathop{\mathbf{c}}}\,\overset{\to }{\mathop{\mathbf{a}}}\,\overset{\to }{\mathop{\mathbf{b}}}\,]}\]                 \[=\frac{[\overset{\to }{\mathop{\mathbf{a}}}\,\overset{\to }{\mathop{\mathbf{b}}}\,\overset{\to }{\mathop{\mathbf{c}}}\,]-[\overset{\to }{\mathop{\mathbf{a}}}\,\overset{\to }{\mathop{\mathbf{b}}}\,\overset{\to }{\mathop{\mathbf{c}}}\,]}{[\overset{\to }{\mathop{\mathbf{c}}}\,\overset{\to }{\mathop{\mathbf{a}}}\,\overset{\to }{\mathop{\mathbf{b}}}\,]}=0\]


You need to login to perform this action.
You will be redirected in 3 sec spinner